Đến nội dung

Hình ảnh

Toán tính tổng,tích Đại Số- Tuyển tập sưu tầm các bài toán từ Mathlinks.ro

- - - - - tuyển tập-sưu tầm.

  • Please log in to reply
Chủ đề này có 67 trả lời

#1
dark templar

dark templar

    Kael-Invoker

  • Hiệp sỹ
  • 3788 Bài viết
Thân chào tất cả các bạn :)

Thay cho lời mở đầu,mình mong các bạn đọc qua topic sau .

Và có lẽ chúng ta nên vào vấn đề chính nào. :)

Bài toán 1:
Hãy tính tích sau : $P = \prod\limits_{k = 0}^n {{{\left( {\cot \frac{x}{{{2^k}}}} \right)}^{{2^k}}}} $
"Do you still... believe in me ?" Sarah Kerrigan asked Jim Raynor - Starcraft II:Heart Of The Swarm.

#2
dark templar

dark templar

    Kael-Invoker

  • Hiệp sỹ
  • 3788 Bài viết

Bài toán 1:
Hãy tính tích sau : $P = \prod\limits_{k = 0}^n {{{\left( {\cot \frac{x}{{{2^k}}}} \right)}^{{2^k}}}} $

Sau đây là lời giải cho bài toán này như đã hứa :)

Lời giải bài toán 1:
Điều kiện xác định :$x \ne {2^k}m\pi $ với $0 \le k \le n$ và $m \in \mathbb{Z}$.

Đặt ${c_n} = \cos \frac{x}{{{2^n}}}$ và ${s_n} = \sin \frac{x}{{{2^n}}}$.

Khi đó P được viết lại dưới dạng sau :
$$\begin{array}{rcl}
P &=& \frac{{\prod\limits_{k = 0}^n c_{k}^{2^{k}}}}{{\prod\limits_{k = 0}^n s_k^{{2^k}}}} = \frac{{\prod\limits_{k = 0}^n c_k^{{2^k}}}}{{\prod\limits_{k = 0}^n {{2^{{2^k}}}} s_{k + 1}^{{2^k}}c_{k + 1}^{{2^k}}}} \\
&=& \frac{{\prod\limits_{k = 0}^n c_k^{{2^k}}}}{{{2^{{2^{n + 1}} - 1}}\prod\limits_{k = 0}^n s_{k + 1}^{{2^k}}\prod\limits_{k = 0}^n c_{k + 1}^{{2^k}}}} \\
\Rightarrow {P^2} &=& \frac{{\prod\limits_{k = 0}^n c_k^{{2^{k + 1}}}}}{{{2^{{2^{n + 2}} - 2}}\prod\limits_{k = 0}^n s_{k + 1}^{{2^{k + 1}}}\prod\limits_{k = 0}^n c_{k + 1}^{{2^{k + 1}}}}}
\end{array}$$

Nhưng $\prod\limits_{k = 0}^n s_{k + 1}^{{2^{k + 1}}} = \frac{{s_{n + 1}^{{2^{n + 1}}}}}{{{s_0}}}\prod\limits_{k = 0}^n s_k^{{2^k}}$ và $\prod\limits_{k = 0}^n c_{k + 1}^{{2^{k + 1}}} = \frac{{c_{n + 1}^{{2^{n + 1}}}}}{{{c_0}}}\prod\limits_{k = 0}^n c_k^{{2^k}}$ nên :


$$\begin{array}{rcl}
{P^2} &=& \frac{{\prod\limits_{k = 0}^n c_k^{{2^{k + 1}}}}}{{{2^{{2^{n + 2}} - 2}}\prod\limits_{k = 0}^n s_k^{{2^k}}\prod\limits_{k = 0}^n c_k^{{2^k}}}}\frac{{{c_0}{s_0}}}{{c_{n + 1}^{{2^{n + 1}}}s_{n + 1}^{{2^{n + 1}}}}}\\
&=& \frac{{\prod\limits_{k = 0}^n c_k^{{2^k}}}}{{{2^{{2^{n + 2}} - 2}}\prod\limits_{k = 0}^n s_k^{{2^k}}}}\frac{{{c_0}{s_0}}}{{c_{n + 1}^{{2^{n + 1}}}s_{n + 1}^{{2^{n + 1}}}}}\\
&=& \frac{P}{{{2^{{2^{n + 2}} - 2}}}}\frac{{{c_0}{s_0}}}{{c_{n + 1}^{{2^{n + 1}}}s_{n + 1}^{{2^{n + 1}}}}}
\end{array}$$

Và nếu $x \ne {2^k}\left( {\frac{\pi }{2} + m\pi } \right)$ với $0 \le k \le n$ và $m \in \mathbb{Z}$ thì $P \neq 0$.

Suy ra $P = \frac{1}{{{2^{{2^{n + 2}} - 2}}}}\frac{{{c_0}{s_0}}}{{c_{n + 1}^{{2^{n + 1}}}s_{n + 1}^{{2^{n + 1}}}}}$

Vậy $P = \frac{{\sin 2x}}{{{2^{{2^{n + 1}} - 1}}{{\left( {\sin \frac{x}{{{2^n}}}} \right)}^{{2^{n + 1}}}}}} \quad \forall x \ne {2^k}m\pi $ với $0 \le k \le n$ và $m \in \mathbb{Z}$.

**********
Mong các bạn có thêm ý kiến hay lời giải khác cho bài toán này. :)

Và... bắt đầu đề mới :

Bài toán 2: Cho dãy ${a_n} = \sum\limits_{i = 1}^n {\frac{1}{{(i - 1)! + i!}}} $.Tính tổng $S = \left( {\sum\limits_{i = 1}^n i {a_i}} \right)$

Bài toán 3: Chứng minh rằng $\sum\limits_{k = 1}^{2n - 1} {\frac{{{{( - 1)}^{k - 1}}}}{{\left( \begin{array}{c}
2n\\
k
\end{array} \right)}}} = \frac{1}{{n + 1}}$

Nào,mời các bạn cùng chém và thảo luận cho ý kiến. :)
"Do you still... believe in me ?" Sarah Kerrigan asked Jim Raynor - Starcraft II:Heart Of The Swarm.

#3
dark templar

dark templar

    Kael-Invoker

  • Hiệp sỹ
  • 3788 Bài viết

Bài toán 2: Cho dãy ${a_n} = \sum\limits_{i = 1}^n {\frac{1}{{(i - 1)! + i!}}} $.Tính tổng $S = \left( {\sum\limits_{i = 1}^n i {a_i}} \right)$

Bài toán 3: Chứng minh rằng $\sum\limits_{k = 1}^{2n - 1} {\frac{{{{( - 1)}^{k - 1}}}}{{\left( \begin{array}{c}
2n\\
k
\end{array} \right)}}} = \frac{1}{{n + 1}}$

Mong chờ cao thủ vô chém mà không có :(

Lời giải bài toán 2:
Ta có :
$$\frac{1}{{(i - 1)! + i!}} = \frac{1}{{(i - 1)!(i + 1)}} = \frac{1}{{i!}} - \frac{1}{{(i + 1)!}}$$

Vậy ${a_n} = 1 - \frac{1}{{(n + 1)!}}$.

Do đó :
$$n{a_n} = n - \frac{n}{{(n + 1)!}} = \left( {\frac{{n(n + 1)}}{2} + \frac{1}{{(n + 1)!}}} \right) - \left( {\frac{{(n - 1)n}}{2} + \frac{1}{{n!}}} \right)$$

Từ đó ta tìm được tổng cần tính là ${\frac{{n(n + 1)}}{2} + \frac{1}{{(n + 1)!}} - 1}$

Lời giải bài toán 3:
Đặt ${a_n} = \sum\limits_{k = 0}^n {\frac{{{{( - 1)}^{k - 1}}}}{{\left( \begin{array}{c}
n\\
k
\end{array} \right)}}} $.Khi đó ta có các hệ thức :

$$\begin{array}{l}
{a_n} = {( - 1)^{n - 1}} + \sum\limits_{k = 0}^{n - 1} {\frac{{n - k}}{n}} \frac{{{{( - 1)}^{k - 1}}}}{{\left( \begin{array}{c}
n - 1\\
k
\end{array} \right)}}\\
\Leftrightarrow {a_n} = {( - 1)^{n - 1}} + \sum\limits_{k = 0}^{n - 1} {\frac{{{{( - 1)}^{k - 1}}}}{{\left( \begin{array}{c}
n - 1\\
k
\end{array} \right)}}} - \sum\limits_{k = 0}^{n - 1} {\frac{k}{n}} \frac{{{{( - 1)}^{k - 1}}}}{{\left( \begin{array}{c}
n - 1\\
k
\end{array} \right)}}
\end{array}$$

Lúc này ${a_n} = {( - 1)^{n - 1}} + {a_{n - 1}} - {b_n}$,với ${b_n} = \sum\limits_{k = 0}^{n - 1} {\frac{k}{n}} \frac{{{{( - 1)}^{k - 1}}}}{{\left( \begin{array}{c}
n - 1\\
k
\end{array} \right)}}$.

Nhưng :

$$\begin{array}{rcl}
{b_n} &=& \sum\limits_{k = 0}^{n - 1} {\frac{{n - 1 - k}}{n}} \frac{{{{( - 1)}^{n - 1 - k - 1}}}}{{\left( \begin{array}{c}
n - 1\\
n - 1 - k
\end{array} \right)}}\\
&=& \sum\limits_{k = 0}^{n - 1} ( \frac{{n - 1}}{n} - \frac{k}{n})\frac{{{{( - 1)}^{n + 1}}{{( - 1)}^{k - 1}}}}{{\left( \begin{array}{c}
n - 1\\
k
\end{array} \right)}}\\
&=& \frac{{n - 1}}{n}{( - 1)^{n + 1}}\sum\limits_{k = 0}^{n - 1} {\frac{{{{( - 1)}^{k - 1}}}}{{\left( \begin{array}{c}
n - 1\\
k
\end{array} \right)}}} - {( - 1)^{n + 1}}\sum\limits_{k = 0}^{n - 1} {\frac{k}{n}} \frac{{{{( - 1)}^{k - 1}}}}{{\left( \begin{array}{c}
n - 1\\
k
\end{array} \right)}}\\
&=& \frac{{n - 1}}{n}{( - 1)^{n + 1}}{a_{n - 1}} - {( - 1)^{n + 1}}{b_n}\\
\Leftrightarrow \left[ {1 + {{( - 1)}^{n + 1}}} \right]{b_n} &=& \frac{{n - 1}}{n}{( - 1)^{n + 1}}{a_{n - 1}}
\end{array}$$

Vậy $a_{n-1}=0$ với mọi $n$ chẵn và ${b_n} = \frac{{n - 1}}{{2n}}{a_{n - 1}}$ với mọi $n$ lẻ.

Quay lại công thức ${a_n} = {( - 1)^{n - 1}} + {a_{n - 1}} - {b_n}$ và xét với $n$ lẻ,ta có:
$$0 = 1 + {a_{n - 1}} - \frac{{n - 1}}{{2n}}{a_{n - 1}} \Rightarrow {a_{n - 1}} = - \frac{{2n}}{{n + 1}},\forall n \quad \text{lẻ}$$

Vậy ${a_{2n}} = - \frac{{2n + 1}}{{n + 1}}$.

Vì ${a_{2n}} = \sum\limits_{k = 0}^{2n} {\frac{{{{( - 1)}^{k - 1}}}}{{\left( \begin{array}{c}
2n\\
k
\end{array} \right)}}} = - 2 + \sum\limits_{k = 1}^{2n - 1} {\frac{{{{( - 1)}^{k - 1}}}}{{\left( \begin{array}{c}
2n\\
k
\end{array} \right)}}} $ nên ta có :

$$\sum\limits_{k = 1}^{2n - 1} {\frac{{{{( - 1)}^{k - 1}}}}{{\left( \begin{array}{c}
2n\\
k
\end{array} \right)}}} = 2 + {a_{2n}} = 2 - \frac{{2n + 1}}{{n + 1}} = \frac{1}{{n + 1}}$$

Do vậy ta có đpcm.

Chú ý :Ta có thể dùng đẳng thức $\frac{1}{{\left( \begin{array}{c}
2n\\
k
\end{array} \right)}} = \frac{{2n + 1}}{{2n + 2}}\left( {\frac{1}{{\left( \begin{array}{c}
2n + 1\\
k
\end{array} \right)}} + \frac{1}{{\left( \begin{array}{c}
2n + 1\\
k + 1
\end{array} \right)}}} \right)$ để giải bài toán này.
"Do you still... believe in me ?" Sarah Kerrigan asked Jim Raynor - Starcraft II:Heart Of The Swarm.

#4
dark templar

dark templar

    Kael-Invoker

  • Hiệp sỹ
  • 3788 Bài viết
Và ... tiếp tục đề mới nào :

Bài toán 4: Cho $a = {e^{(i2\pi )/n}}$.Tính $\sum\limits_{k = 0}^{n - 1} {{{(1 + {a^k})}^n}} $.

Bài toán 5: Cho $n$ là 1 số nguyên dương ,tính tổng ${\cal A} = \sum\limits_{i = 1}^n ( - 1{)^k}(n + k)!(n - k)!$
"Do you still... believe in me ?" Sarah Kerrigan asked Jim Raynor - Starcraft II:Heart Of The Swarm.

#5
hxthanh

hxthanh

    Tín đồ $\sum$

  • Hiệp sỹ
  • 3915 Bài viết
Bài toán 4: Có vẻ không khả thi lắm :)

Chú ý rằng: $a^n=1$ và $\sum_{k=0}^{n-1} a^k=0$

Giả sử $n$ là số nguyên tố! ( :P cho dễ), khi đó:
$\sum_{k=0}^{n-1} (1+a^k)^n=\sum_{k=0}^{n-1}\sum_{j=0}^n {n\choose j}a^{kj}$
$\quad=\sum_{j=0}^{n}{n\choose j}\sum_{k=0}^{n-1}a^{kj}=2n+\sum_{j=1}^{n-1}{n\choose j}\sum_{k=0}^{n-1}a^{kj}\quad$ (tách riêng $j=0$ và $j=n$)
Do $\text{gcd}(kj,n)=1$ với $0<k,j<n$ nên ứng với mỗi $j$ thì $0,j,2j,...,(n-1)j$ lập nên một hệ thặng dư đầy đủ module $n$
Hay nói cách khác:
hai tập này $\left\{a^{kj}\bigg|k=\overline{0,n-1}\right\}\quad$ và $\{1,a,a^2,...,a^{n-1}\}$ trùng nhau!

Nghĩa là: $\sum_{k=0}^{n-1} (1+a^k)^n=2n\quad$ với $n$ nguyên tố!
Vấn đề


#6
dark templar

dark templar

    Kael-Invoker

  • Hiệp sỹ
  • 3788 Bài viết

Bài toán 4: Có vẻ không khả thi lắm :)

Chú ý rằng: $a^n=1$ và $\sum_{k=0}^{n-1} a^k=0$

Giả sử $n$ là số nguyên tố! ( :P cho dễ), khi đó:
$\sum_{k=0}^{n-1} (1+a^k)^n=\sum_{k=0}^{n-1}\sum_{j=0}^n {n\choose j}a^{kj}$
$\quad=\sum_{j=0}^{n}{n\choose j}\sum_{k=0}^{n-1}a^{kj}=2n+\sum_{j=1}^{n-1}{n\choose j}\sum_{k=0}^{n-1}a^{kj}\quad$ (tách riêng $j=0$ và $j=n$)

Hic,anh Thanh ơi,anh chỉ cần tách $\sum_{j=1}^{n-1}{n\choose j}\sum_{k=0}^{n-1}a^{kj}=\sum_{j=1}^{n-1}\binom{n}{j}\frac{a^{nj}-1}{a^{j}-1}=0$ do $a^{n}=1$ là được rồi mà :mellow:
"Do you still... believe in me ?" Sarah Kerrigan asked Jim Raynor - Starcraft II:Heart Of The Swarm.

#7
hxthanh

hxthanh

    Tín đồ $\sum$

  • Hiệp sỹ
  • 3915 Bài viết

Hic,anh Thanh ơi,anh chỉ cần tách $\sum_{j=1}^{n-1}{n\choose j}\sum_{k=0}^{n-1}a^{kj}=\sum_{j=1}^{n-1}\binom{n}{j}\frac{a^{nj}-1}{a^{j}-1}=0$ do $a^{n}=1$ là được rồi mà :mellow:

Ừ nhỉ? Già rồi nên "lẩn thẩn" đó mà! :luoi:

Vậy tổng đó bằng $\boxed{2n}$ Phúc nhỉ? :)

@Dark templar: Đúng rồi đó anh :) Giờ chỉ còn cần "xơi" bài toán 5 nữa thôi :)

Bài viết đã được chỉnh sửa nội dung bởi dark templar: 17-03-2013 - 18:33


#8
hxthanh

hxthanh

    Tín đồ $\sum$

  • Hiệp sỹ
  • 3915 Bài viết

Bài toán 5: Cho $n$ là 1 số nguyên dương ,tính tổng ${\cal A} = \sum\limits_{k = 1}^n ( - 1{)^k}(n + k)!(n - k)!$

Buồn buồn, lôi bài này ra xử! :D

 

Ta có: ${\cal A}=\sum_{k=1}^n (-1)^k(n+k)!(n-k)!=\sum_{k=1}^n \dfrac{(-1)^k(n+k)!(n-k)![(n+1+k)+(n+1-k)]}{2(n+1)}$

 

$\qquad=\dfrac{1}{2(n+1)}\sum_{k=1}^n\left[(-1)^k(n+1+k)!(n-k)!-(-1)^{k-1}(n+k)!(n+1-k)!\right]$

$\qquad=\dfrac{1}{2(n+1)}\sum_{k=1}^n\Delta\left[(-1)^{k-1}(n+k)!(n+1-k)!\right]$

$\qquad=\dfrac{1}{2(n+1)}(-1)^{k-1}(n+k)!(n+1-k)!\bigg|_{k=1}^{n+1}$

$\qquad=\dfrac{(-1)^n(2n+1)!-n!(n+1)!}{2(n+1)}$



#9
dark templar

dark templar

    Kael-Invoker

  • Hiệp sỹ
  • 3788 Bài viết

Tiếp tục up đề cho anh Thanh và các bạn :)

 

Bài toán 6: Tính $S = 1 + \frac{{cosx}}{{co{s^1}x}} + \frac{{cos(2x)}}{{co{s^2}x}} + ... + \frac{{cos(nx)}}{{co{s^n}x}}$ với $\sin 2x \neq 0$.

 

Bài toán 7: Tính tổng $\sum\limits_{r = 1}^n ( {2^{r - 1}}\cos r\theta )$.


Bài viết đã được chỉnh sửa nội dung bởi dark templar: 01-06-2013 - 21:58
Chèn link !

"Do you still... believe in me ?" Sarah Kerrigan asked Jim Raynor - Starcraft II:Heart Of The Swarm.

#10
hxthanh

hxthanh

    Tín đồ $\sum$

  • Hiệp sỹ
  • 3915 Bài viết

Bài toán 6: Tính $S = 1 + \frac{{cosx}}{{co{s^1}x}} + \frac{{cos(2x)}}{{co{s^2}x}} + ... + \frac{{cos(nx)}}{{co{s^n}x}}$ với $\sin 2x \neq 0$.

Bài toán này, một lần nữa sẽ được giải quyết đẹp với Sai phân cấp 2 :D

 

Ta có: $S=\sum_{k=0}^n \dfrac{\cos(kx)}{\cos^k(x)}$

 

Sai phân cấp 1: $\Delta\left[\dfrac{\cos(kx)}{\cos^k(x)}\right]=\dfrac{\cos((k+1)x)}{\cos^{k+1}(x)}-\dfrac{\cos(kx)}{\cos^k(x)}=\dfrac{-\sin(x)\sin(kx)}{\cos^{k+1}(x)}$

Sai phân cấp 2:

$\Delta^2\left[\dfrac{\cos(kx)}{\cos^k(x)}\right]=\Delta\left[\dfrac{-\sin(x)\sin(kx)}{\cos^{k+1}(x)}\right]=-\sin(x)\left[\dfrac{\sin((k+1)x)}{\cos^{k+2}(x)}-\dfrac{\sin(kx)}{\cos^{k+1}(x)}\right]$

$\qquad\qquad\qquad=-\dfrac{\sin^2(x)}{\cos^2(x)}\cdot\dfrac{\cos(kx)}{\cos^k(x)}$

 

Như vậy ta có:

 

$\begin{align*} -\dfrac{\sin^2(x)}{\cos^2(x)}S&=\sum_{k=0}^n\Delta^2\left[\dfrac{\cos(kx)}{\cos^k(x)}\right]\\&=\left.\Delta\left[\dfrac{\cos(kx)}{\cos^k(x)}\right]\right|_{k=0}^{n+1}\\ &=\dfrac{\cos((n+2)x)}{\cos^{n+2}(x)}-\dfrac{\cos((n+1)x)}{\cos^{n+1}(x)}-\dfrac{\cos(x)}{\cos(x)}+\dfrac{\cos(0.x)}{\cos^0(x)}\\&=\dfrac{-\sin(x)\sin((n+1)x)}{\cos^{n+2}(x)}\end{align*}$

 

Từ đó ta có: $\quad S=\dfrac{\sin((n+1)x)}{\sin(x)\cos^n(x)}$

 

________________________________

@hxthanh: Bài 7 thì quá dễ với SPTP rồi :))



#11
dark templar

dark templar

    Kael-Invoker

  • Hiệp sỹ
  • 3788 Bài viết

@anh Thanh :Cả 2 bài em đều chém bằng SP cấp 1. 

 

Bài toán 6: 

Xét số hạng tổng quát :

 

\[\begin{array}{rcl}\frac{{\cos kx}}{{{{\cos }^k}x}} &=& \frac{{\cos kx\sin x}}{{{{\cos }^k}x\sin x}}\\&=& \frac{1}{2}\left[ {\frac{{\sin \left( {k + 1} \right)x}}{{{{\cos }^k}x\sin x}} - \frac{{\sin \left( {k - 1} \right)x}}{{{{\cos }^k}x\sin x}}} \right]\\&=& \frac{1}{2}\left[ {\frac{{\sin \left( {k + 1} \right)x}}{{{{\cos }^k}x\sin x}} - \frac{{\sin kx\cos x - \cos kx\sin x}}{{{{\cos }^k}x\sin x}}} \right]\\&=& \frac{1}{2}\left[ {\frac{{\sin \left( {k + 1} \right)x}}{{{{\cos }^k}x\sin x}} - \frac{{\sin kx}}{{{{\cos }^{k - 1}}x\sin x}}} \right] + \frac{1}{2}\frac{{\cos kx}}{{{{\cos }^k}x}}\\\Rightarrow \frac{{\cos kx}}{{{{\cos }^k}x}} &=& \Delta \left[ {\frac{{\sin kx}}{{{{\cos }^{k - 1}}x\sin x}}} \right]\end{array}\]

 

Từ đó ta có:

\[S = \sum\limits_{k = 0}^n {\frac{{\cos kx}}{{{{\cos }^k}x}}}  = \left[ {\frac{{\sin kx}}{{{{\cos }^{k - 1}}x\sin x}}} \right]_{k = 0}^{n + 1} = \frac{{\sin \left( {n + 1} \right)x}}{{{{\cos }^n}x\sin x}}\]

 

**********

Bài toán 7:

Đặt $S_{n}=\sum_{r=1}^{n}2^{r-1}\cos r\theta$.

 

Dễ thấy :

 

\[\begin{array}{l}{2^{r - 1}} = \Delta \left[ {{2^{r - 1}}} \right]\\\Delta \left[ {\cos r\theta } \right] =  - 2\sin \frac{\theta }{2}\sin \left( {r + \frac{1}{2}} \right)\theta \end{array}\]
 
Nên theo SPTP :

\[\begin{array}{rcl}{S_n} &=& \left[ {{2^{r - 1}}\cos r\theta } \right]_{r = 1}^{n + 1} + \sin \frac{\theta }{2}\sum\limits_{r = 1}^n {{2^{r + 1}}} \sin \left( {r + \frac{1}{2}} \right)\theta \\&=& \left[ {{2^{r - 1}}\cos r\theta } \right]_{r = 1}^{n + 1} + \sin \frac{\theta }{2}\sum\limits_{r = 1}^n {\Delta \left[ {{2^{r + 1}}} \right]\sin \left( {r + \frac{1}{2}} \right)\theta } \\&=& \left[ {{2^{r - 1}}\cos r\theta } \right]_{r = 1}^{n + 1} + \sin \frac{\theta }{2}\left[ {{2^{r + 1}}\sin \left( {r + \frac{1}{2}} \right)\theta } \right]_{r = 1}^{n + 1}\\&-& \sin \frac{\theta }{2}\sum\limits_{r = 1}^n {{2^{r + 2}}\Delta \left[ {\sin \left( {r + \frac{1}{2}} \right)\theta } \right]} \\&=& {2^{n + 2}}\sin \frac{\theta }{2}\sin \frac{{2n + 3}}{2}\theta  - 4\sin \frac{\theta }{2}\sin \frac{{3\theta }}{2} + \left[ {{2^{r - 1}}\cos r\theta } \right]_{r = 1}^{n + 1}\\&-& {\sin ^2}\frac{\theta }{2}\sum\limits_{r = 1}^n {{2^{r + 3}}\cos \left( {r + 1} \right)\theta } \\&=& {2^{n + 2}}\sin \frac{\theta }{2}\sin \frac{{2n + 3}}{2}\theta  - 4\sin \frac{\theta }{2}\sin \frac{{3\theta }}{2} + \left[ {{2^{r - 1}}\cos r\theta } \right]_{r = 1}^{n + 1}\\&-& 8{\sin ^2}\frac{\theta }{2}{S_n} - 8{\sin ^2}\frac{\theta }{2}\left[ {{2^{r - 1}}\cos r\theta } \right]_{r = 1}^{n + 1}\end{array}\]
 
Từ đó dễ dàng tính ra được $S_{n}$.
 
:)

Bài viết đã được chỉnh sửa nội dung bởi dark templar: 24-03-2013 - 10:39

"Do you still... believe in me ?" Sarah Kerrigan asked Jim Raynor - Starcraft II:Heart Of The Swarm.

#12
dark templar

dark templar

    Kael-Invoker

  • Hiệp sỹ
  • 3788 Bài viết

Và sau đây là lời giải chính thức cho 2 bài toán 6 và 7.Cả 2 lời giải đều sử dụng số phức :)

 

Lời giải bài toán 6:

$\frac{{\cos kx}}{{{{\cos }^k}x}}$ là phần thực của số phức ${\left( {\frac{{{e^{ix}}}}{{\cos x}}} \right)^k}$.

 

Vậy ta có 1 CSN với S là phần thực của $\frac{{{{\left( {\frac{{{e^{ix}}}}{{\cos x}}} \right)}^{n + 1}} - 1}}{{\left( {\frac{{{e^{ix}}}}{{\cos x}}} \right) - 1}}$ hay $\frac{1}{{i\tan x}}\left( {\frac{{{e^{i(n + 1)x}}}}{{{{\cos }^{n + 1}}x}} - 1} \right)$.

 

Từ đó ta tìm được phần thực hay tổng S là $\frac{{\sin (n + 1)x}}{{\sin x{{\cos }^n}x}}$.

 

Lời giải bài toán 7:

Tổng cần tìm là phần thực của $z = \sum\limits_{r = 1}^n {{2^{r - 1}}} {e^{ir\theta }} = \frac{1}{2}\sum\limits_{r = 1}^n ( 2{e^{i\theta }}{)^r} = \frac{1}{2}\frac{{{{(2{e^{i\theta }})}^{n + 1}} - 2{e^{i\theta }}}}{{2{e^{i\theta }} - 1}}$.

 

Ta có :

 

$$\begin{array}{l}z = \frac{1}{{2(5 - 4\cos \theta )}}({(2{e^{i\theta }})^{n + 1}} - 2{e^{i\theta }})(2{e^{ - i\theta }} - 1)\\\Leftrightarrow z = \frac{1}{{2(5 - 4\cos \theta )}}({2^{n + 2}}{e^{in\theta }} - 4 - {2^{n + 1}}{e^{i(n + 1)\theta }} + 2{e^{i\theta }})\end{array}$$
 
Từ đây ta tìm được tổng là $\frac{{{2^{n + 1}}\cos n\theta  - {2^n}\cos (n + 1)\theta  + \cos \theta  - 2}}{{5 - 4\cos \theta }}$.
 
**********
Đề mới :
 
Bài toán 8: Tính giá trị của $S = \sum\limits_{k = 1}^n {{{\cos }^2}} \left( {\frac{{2k\pi }}{n}} \right)$.
 
Tổng quát thì ta có kết quả sau :$\sum\limits_{k = 1}^n {{{\cos }^p}} \frac{{2k\pi }}{n} = \frac{n}{{{2^p}}}\sum\limits_{k = 0,n|2k - p}^p {\left( \begin{array}{c}p\\k\end{array} \right)} $.
 
Bài toán 9: Nếu ${\left( {1 + {x^{2005}} + {x^{2006}} + {x^{2007}}} \right)^{2008}} = {a_0} + {a_1}{x^1} + {a_2}{x^2} + ........ + {a_n}{x^n}$,tính giá trị của tổng ${a_0} - \frac{1}{3}{a_1} - \frac{1}{3}{a_2} + {a_3} - \frac{1}{3}{a_4} - \frac{1}{3}{a_5} + {a_6} - .................$.

 


"Do you still... believe in me ?" Sarah Kerrigan asked Jim Raynor - Starcraft II:Heart Of The Swarm.

#13
hxthanh

hxthanh

    Tín đồ $\sum$

  • Hiệp sỹ
  • 3915 Bài viết

Xem ra anh em mình tiếp tục "tự biên tự diễn" vậy! :D

 

Anh đăng ký cả hai bài mới này! :luoi: (hơi tham!)

 

@dark templar: Phải rủ cu Hân vô cho vui nữa :))


Bài viết đã được chỉnh sửa nội dung bởi dark templar: 24-03-2013 - 11:30


#14
hxthanh

hxthanh

    Tín đồ $\sum$

  • Hiệp sỹ
  • 3915 Bài viết

Đề mới :

 
Bài toán 8: Tính giá trị của $S = \sum\limits_{k = 1}^n {{{\cos }^2}} \left( {\frac{{2k\pi }}{n}} \right)$.
 
Tổng quát thì ta có kết quả sau :$\sum\limits_{k = 1}^n {{{\cos }^p}} \frac{{2k\pi }}{n} = \frac{n}{{{2^p}}}\sum\limits_{k = 0,n|2k - p}^p {\left( \begin{array}{c}p\\k\end{array} \right)} $.

Ta có: $\cos\left(\frac{2k\pi}{n}\right)=\dfrac{e^{\frac{2k\pi i}{n}}+e^{-\frac{2k\pi i}{n}}}{2}$

Do đó:

$\quad\sum_{k=1}^n \cos^p\left(\frac{2k\pi}{n}\right)=\dfrac{1}{2^p}\sum_{k=1}^n \left(e^{\frac{2k\pi i}{n}}+e^{-\frac{2k\pi i}{n}}\right)^p$

$=\dfrac{1}{2^p}\sum_{k=1}^n\sum_{j=0}^p {p\choose j} e^{\frac{(p-2j)2k\pi i}{n}}$

$=\dfrac{1}{2^p}\sum_{k=1}^n\sum_{j=0}^p {p\choose j} \left[\cos\left(\frac{(p-2j)2k\pi}{n}\right)+i\sin\left(\frac{(p-2j)2k\pi}{n}\right)\right]$

 

Lưu ý rằng: $\quad\cos\left(\frac{2k\pi}{n}\right)\in \mathbb R,\quad\forall k\in \mathbb N$

nên tổng phía trên được lấy sao cho các thành phần ảo phải bằng $0$ với mọi $k$

Nghĩa là: $\quad\sin\left(\frac{(p-2j)2k\pi}{n}\right)=0\quad \forall k\Rightarrow n\mid p-2j$

Với điều kiện đó thì: $\quad\cos\left(\frac{(p-2j)2k\pi}{n}\right)=1\quad \forall k$

 

Suy ra $\quad\sum_{k=1}^n \cos^p\left(\frac{2k\pi}{n}\right)=\dfrac{n}{2^p}\sum_{\substack{ 0\le j\le p\\ n\mid p-2j}}{p\choose j}$



#15
robin997

robin997

    Thượng sĩ

  • Thành viên
  • 207 Bài viết

Bài toán 8: Tính giá trị của $S = \sum\limits_{k = 1}^n {{{\cos }^2}} \left( {\frac{{2k\pi }}{n}} \right)$.
 
Tổng quát thì ta có kết quả sau :$\sum\limits_{k = 1}^n {{{\cos }^p}} \frac{{2k\pi }}{n} = \frac{n}{{{2^p}}}\sum\limits_{k = 0,n|2k - p}^p {\left( \begin{array}{c}p\\k\end{array} \right)} $.
 
:mellow: ... hai anh em tự biên tự diễn
Một hướng khác cho Bài toán 8:
-Với $e^{i\frac{2k\pi }{n}}=cos(\frac{2k\pi }{n})+isin(\frac{2k\pi }{n})$

Nên nếu lấy $k$ chạy từ $1$ đến $n$, ta được $n$ nghiệm phân biệt của phương trình:
$z^n=1\Leftrightarrow z^n-1=0$
-Ta kí hiệu như sau:
$_p\delta _n=\sum_{k=1}^n{cos(p\frac{2k\pi }{n})}$
-Theo định lý Viète, ta có:
$\begin{cases}
& \Delta_1e^{i\frac{2k\pi }{n}}=\sum_{k=1}^n{cos(\frac{2k\pi }{n})}+i(..)=_1\delta^n+i(..)=0 (n>1) \\
& \Delta_2e^{i\frac{2k\pi }{n}}=0\\
& .... \\
& \Delta_ne^{i\frac{2k\pi }{n}}=\prod_{k=1}^ne^{i\frac{2k\pi }{n}}=(-1)^{n+1}
\end{cases}$ ( $_1\delta_1=1$ )
$\rightarrow Re\Delta_1\left(e^{i\frac{2k\pi }{n}}\right)^p = _p\delta_n$
Dễ có $\forall n>p$: $\Delta_1\left(e^{i\frac{2k\pi }{n}}\right)^p=0\Rightarrow Re\Delta_1\left(e^{i\frac{2k\pi }{n}}\right)^p = _p\delta_n=0 $
Theo đó, ta có:
$S=\sum_{k=1}^n{cos^2(\frac{2k\pi }{n})}=\sum_{k=1}^n\frac{1}{2}\left({cos(2\frac{2k\pi }{n})+1}\right)=\frac{1}{2} {_2\delta_n}+\frac{n}{2}=\begin{cases}
&\frac{1}{2} {_2\delta_1}+\frac{1}{2}=1 (n=1)\\ &\frac{1}{2} {_2\delta_2}+\frac{2}{2}=2 (n=2)\\ & \frac{n}{2} (n>2) \end{cases}$
-Tổng quát bài toán trong trường hợp $n>p$ :
$\sum_{k=1}^n{cos^p(\frac{2k\pi }{n})}=\sum_{k=1}^n{\frac{1}{2^p} }\sum_{e=0}^pC_p^eCos\left((p-2e)\frac{2k\pi }{n}\right)=\frac{1}{2^p}\sum_{e=0}^pC_p^e {(_{p-2e}\delta_n)}=\begin{cases}
& 0 (p \not{\vdots } 2) \\
& \frac{n}{2^p} C_p^{\frac{p}{2}}  (p\vdots 2)
\end{cases}$ 
....Cách này làm cho $n\leq p$ được không mấy anh?


Bài viết đã được chỉnh sửa nội dung bởi robin997: 26-03-2013 - 15:29

^^~

#16
robin997

robin997

    Thượng sĩ

  • Thành viên
  • 207 Bài viết

Bài toán 9: Nếu ${\left( {1 + {x^{2005}} + {x^{2006}} + {x^{2007}}} \right)^{2008}} =f(x) = {a_0} + {a_1}{x^1} + {a_2}{x^2} + ........ + {a_n}{x^n}$,tính giá trị của tổng ${a_0} - \frac{1}{3}{a_1} - \frac{1}{3}{a_2} + {a_3} - \frac{1}{3}{a_4} - \frac{1}{3}{a_5} + {a_6} - .................$.


-Tổng cần tính:
$S=\sum_{i\vdots 3}a_i -\frac{1}{3}\sum_{i\not{\vdots} 3}a_i=\frac{1}{3}( 4 \sum_{i\vdots 3}a_i - \sum a_i)\\
= \frac{1}{3}( \frac{4}{3} ( f(1)+ f(e^{i \frac{2 \pi}{3}})+f(e^{i 2\frac{2 \pi}{3}}))-f(1))=\frac{1}{9} (4^{2008} +4.2.1^{2008})=\frac{4^{2008}+8}{9}$


Bài viết đã được chỉnh sửa nội dung bởi robin997: 26-03-2013 - 16:03

^^~

#17
hxthanh

hxthanh

    Tín đồ $\sum$

  • Hiệp sỹ
  • 3915 Bài viết

Trong khi chờ đợi dark templar post đề mới, mời các bạn cùng "giải trí" với bài toán sau:

 

Bài toán (*):

Với $1\le n\in\mathbb N$

Tính tổng $S=\sum_{k=1}^n k\left\lfloor\sqrt k\right\rfloor$

 

Bài toán (**):

Với $a\ne 1<p<n;\;\;n,p\in\mathbb N$

Tính tổng $S=\sum_{k=1}^na^k\left\lfloor\dfrac{k}{p}\right\rfloor$

 

__________________________

 

P/s: ê sắc rồi :(


Bài viết đã được chỉnh sửa nội dung bởi dark templar: 31-03-2013 - 21:04


#18
dark templar

dark templar

    Kael-Invoker

  • Hiệp sỹ
  • 3788 Bài viết

Bài toán 8: Tính giá trị của $S = \sum\limits_{k = 1}^n {{{\cos }^2}} \left( {\frac{{2k\pi }}{n}} \right)$.

 
Tổng quát thì ta có kết quả sau :$\sum\limits_{k = 1}^n {{{\cos }^p}} \frac{{2k\pi }}{n} = \frac{n}{{{2^p}}}\sum\limits_{k = 0,n|2k - p}^p {\left( \begin{array}{c}p\\k\end{array} \right)} $.
 
Bài toán 9: Nếu ${\left( {1 + {x^{2005}} + {x^{2006}} + {x^{2007}}} \right)^{2008}} = {a_0} + {a_1}{x^1} + {a_2}{x^2} + ........ + {a_n}{x^n}$,tính giá trị của tổng ${a_0} - \frac{1}{3}{a_1} - \frac{1}{3}{a_2} + {a_3} - \frac{1}{3}{a_4} - \frac{1}{3}{a_5} + {a_6} - .................$.

@anh Thanh: 2 bài của anh mà đem "giải trí" thì... :wacko: Em chỉ thấy có mỗi bài đầu là khả dĩ làm được thôi :(

@robin997: Bài 9 lúc em giải bằng định lý RUF thì nhớ ghi chú nhé :)

 

**********

Lời giải bài toán 8:

Ta có :

 

$$\begin{array}{rcl}4S &=& \sum\limits_{k = 1}^n {{{\left( {{e^{\frac{{2ki\pi }}{n}}} + {e^{\frac{{ - 2ki\pi }}{n}}}} \right)}^2}} \\&=& \sum\limits_{k = 1}^n {\left( {{e^{\frac{{4ki\pi }}{n}}} + {e^{\frac{{ - 4ki\pi }}{n}}} + 2} \right)} \\&=& 2n + \sum\limits_{k = 1}^n {{{\left( {{e^{\frac{{4i\pi }}{n}}}} \right)}^k}}  + \sum\limits_{k = 1}^n {{{\left( {{e^{\frac{{ - 4i\pi }}{n}}}} \right)}^k}}\end{array}$$
 
Nếu $n=1$ hay $n=2$ thì ${e^{\frac{{4i\pi }}{n}}} = {e^{ - \frac{{4i\pi }}{n}}} = 1$,$4S = 2n + n + n$ và $S=n$.
 
Nếu $n \ne 1,n \ne 2$ thì ${e^{\frac{{4i\pi }}{n}}} \ne 1$ và do đó $\sum\limits_{k = 1}^n {{{\left( {{e^{\frac{{4i\pi }}{n}}}} \right)}^k}}  = {e^{\frac{{4i\pi }}{n}}}\frac{{{{\left( {{e^{\frac{{4i\pi }}{n}}}} \right)}^n} - 1}}{{{e^{\frac{{4i\pi }}{n}}} - 1}} = 0$.
 
Cách làm cũng được thực hiện tương tự cho tổng $\sum\limits_{k = 1}^n {{{\left( {{e^{\frac{{ - 4i\pi }}{n}}}} \right)}^k}} $.
 
Vậy $4S = 2n \Leftrightarrow S = \frac{n}{2}$.
 
Từ đó giá trị của tổng cần tìm là $S = \left\{ \begin{array}{l}n\left( {n \in \left\{ {1;2} \right\}} \right)\\\frac{n}{2}\left( {n > 2} \right)\end{array} \right.$.

 

Lời giải cho bài toán 9:

Đặt $P(x) = \sum\limits_{k = 1}^n {{a_k}{x^k}} $,$j = {e^{i\frac{{2\pi }}{3}}}$.Khi đó ta có :

 

$$\begin{array}{l}P(1) + P(j) + P({j^2}) = 3\sum {a_{3k}}\\P(1) + jP(j) + {j^2}P({j^2}) = 3\sum {a_{3k + 2}}\\P(1) + {j^2}P(j) + jP({j^2}) = 3\sum {a_{3k + 1}}\end{array}$$
 
Suy ra tổng cần tìm là $S = \sum {a_{3k}} - \frac{1}{3}\sum {a_{3k + 1}} - \frac{1}{3}\sum {a_{3k + 2}}$.
 
Và:

$$\begin{array}{rcl}9S &=& 3(P(1) + P(j) + P({j^2})) - (P(1) + {j^2}P(j) + jP({j^2}))\\&-& (P(1) + jP(j) + {j^2}P({j^2}))\\&=& P(1) + 4P(j) + 4P({j^2})\end{array}$$
 
Với $P(x) = {(1 + {x^{2005}} + {x^{2006}} + {x^{2007}})^{2008}}$,ta có được $P(1) = {4^{2008}};P(j) = P({j^2}) = 1$.
 
Từ đó thì $\boxed{S = \frac{{8 + {4^{2008}}}}{9}}$.
 
**********
Đề mới:
 
Bài toán 10: Chứng minh rằng $\frac{3}{{1.2.4}} + \frac{4}{{2.3.5}} + \frac{5}{{3.4.6}} + ....... + \frac{{(n + 2)}}{{n.(n + 1).(n + 3)}} = \frac{1}{6}\left[ {\frac{{29}}{6} - \frac{4}{{n - 1}}\; - \frac{1}{{n + 2}} - \frac{1}{{n + 3}}} \right]$
 
Bài toán 11: Tính giá trị của tổng $E = \sum\limits_{k = 1}^n {\log } \frac{{(2k + 7)(2k + 5)}}{{k(k + 1)}}.\log \frac{{(2k + 7)k}}{{(2k + 5)(k + 1)}}$.

__________________________________

hxthanh@to Phúc: Bài 10 em để tính tổng thì hay hơn là CM (Vì chứng minh có thể dùng quy nạp được!)

Bài 11 chắc là logarithm tự nhiên theo cách viết quốc tế nhỉ? (thật ra không quan trọng cơ số)

 

Bài (*) phải dùng kỹ thuật phân đoạn ... theo hàm số :P

Bài (**) giải được với SPTP :))

 

@dark templar: Bài 11 thì là log cơ số tự nhiên đấy anh,mà cơ số không quan trọng gì đâu anh :)

Bài (*) em nghĩ nguyên ngày nay mà ko ra,cứ tưởng đã giải được mà tưởng bở ... :(

Bài (**) SPTP á  :ohmy:

 

@hxthanh

Spoiler


Bài viết đã được chỉnh sửa nội dung bởi hxthanh: 01-04-2013 - 18:12
Nhận xét!

"Do you still... believe in me ?" Sarah Kerrigan asked Jim Raynor - Starcraft II:Heart Of The Swarm.

#19
hxthanh

hxthanh

    Tín đồ $\sum$

  • Hiệp sỹ
  • 3915 Bài viết

Trong khi chờ đợi dark templar post đề mới, mời các bạn cùng "giải trí" với bài toán sau:
 
Bài toán (*):
Với $1\le n\in\mathbb N$
Tính tổng $S=\sum_{k=1}^n k\left\lfloor\sqrt k\right\rfloor$
 
Bài toán (**):
Với $a\ne 1<p<n;\;\;n,p\in\mathbb N$
Tính tổng $S=\sum_{k=1}^na^k\left\lfloor\dfrac{k}{p}\right\rfloor$
 
__________________________
 
P/s: ê sắc rồi :(

 
Lời giải bài toán (*)
 
Thực ra bài này chỉ cần biết cách chia đoạn là tính được :D
Đặt $\boxed{\;m=\left\lfloor\sqrt n\right\rfloor\;}$
Ta sẽ chia tổng cần tính $S=\sum_{k=1}^n k\left\lfloor\sqrt k\right\rfloor$ thành $m$ đoạn như sau:
(Phân đoạn đa thức)
 
Với $m-1$ đoạn $[p^2,(p+1)^2-1]\quad(1\le p\le m-1)$ đoạn cuối là $[m^2,n]$
Ta có:
$S=\sum_{p=1}^{m-1}\sum_{k=p^2}^{(p+1)^2-1}k\left\lfloor\sqrt k\right\rfloor+\sum_{k=m^2}^n k\left\lfloor\sqrt k\right\rfloor$
$\quad=\qquad A\qquad\qquad+\qquad B$
 
Xét tổng $B$
Ta có:
$B=\sum_{k=m^2}^n k\left\lfloor\sqrt k\right\rfloor=\sum_{k=m^2}^n km$
$\quad = \sum_{k=1}^n km - \sum_{k=1}^{m^2-1} km\quad$ (Hiệu chung gốc)
$\quad = \dfrac{m.n(n+1)}{2}-\dfrac{m^3(m^2-1)}{2}$
 
Xét tổng $A$
Ta có: với mỗi $k$ trên đoạn $p^2\le k\le (p+1)^2-1$ thì ta có $\left\lfloor\sqrt k\right\rfloor=p$
Như vậy:
$A=\sum_{p=1}^{m-1}\sum_{k=p^2}^{(p+1)^2-1}kp$
$\quad = \sum_{p=1}^{m-1}p\left(\sum_{k=1}^{(p+1)^2-1}k-\sum_{k=1}^{p^2-1}k\right)\quad$ (Hiệu chung gốc)
$\quad = \sum_{p=1}^{m-1}p\left(\dfrac{[(p+1)^2-1](p+1)^2}{2}-\dfrac{(p^2-1)p^2}{2}\right)$
$\quad = \sum_{p=1}^{m-1}p\Delta\left[\dfrac{(p^2-1)p^2}{2}\right]$
$\quad = \left.\dfrac{p^3(p^2-1)}{2}\right|_{p=1}^m-\sum_{p=1}^{m-1}\dfrac{[(p+1)^2-1](p+1)^2}{2}$
$\quad = \dfrac{m^3(m^2-1)}{2}-\sum_{p=1}^{m}\dfrac{(p^2-1)p^2}{2}$
 
Do đó:
$S=A+B=\dfrac{m.n(n+1)}{2}-\sum_{p=1}^{m}\dfrac{(p^2-1)p^2}{2}$
Bỏ qua bước tính cái tổng này, ta có luôn:
 
$\boxed{\;S=\dfrac{m.n(n+1)}{2}-\dfrac{m(m-1)(m+1)(m+2)(2m+1)}{20}\;}$
 
Lời giải bài toán (**)
 
Hy vọng với gợi ý dùng SPTP, thì dark templar sẽ tự giải quyết được bài này trước khi xem đáp án ở dưới

Spoiler



#20
hxthanh

hxthanh

    Tín đồ $\sum$

  • Hiệp sỹ
  • 3915 Bài viết


**********
Đề mới:
 
Bài toán 10: Chứng minh rằng $\frac{3}{{1.2.4}} + \frac{4}{{2.3.5}} + \frac{5}{{3.4.6}} + ....... + \frac{{(n + 2)}}{{n.(n + 1).(n + 3)}} = \frac{1}{6}\left[ {\frac{{29}}{6} - \frac{4}{{n + 1}}\; - \frac{1}{{n + 2}} - \frac{1}{{n + 3}}} \right]$
 
Bài toán 11: Tính giá trị của tổng $E = \sum\limits_{k = 1}^n {\log } \frac{{(2k + 7)(2k + 5)}}{{k(k + 1)}}.\log \frac{{(2k + 7)k}}{{(2k + 5)(k + 1)}}$.

Tại sao 2 bài này mà bị "ế"! :)

Lời giải bài số 11 của hxthanh

Ta có:

$\begin{align*}E&=\sum_{k=1}^n \log\frac{(2k+7)(2k+5)}{k(k+1)}.\log\frac{(2k+7)k}{(2k+5)(k+1)}\\&=\sum_{k=1}^n\left(\log\frac{2k+7}{k+1}+\log\frac{2k+5}{k}\right)\left(\log\frac{2k+7}{k+1}-\log\frac{2k+5}{k}\right)\\&=\sum_{k=1}^n\left(\log^2\frac{2k+7}{k+1}-\log^2\frac{2k+5}{k}\right)\\&=\sum_{k=1}^n\Delta\left(\log^2\frac{2k+5}{k}\right)\\&=\left.\log^2\frac{2k+5}{k}\right|_{k=1}^{n+1}\\&=\log^2\frac{2n+7}{n+1}-\log^2(7)\\&=\log\frac{14n+49}{n+1}.\log\frac{2n+7}{7n+7}\end{align*}$

__________________________________

Lời giải bài số 10 của hxthanh

Tổng cần tính:

$\begin{align*}S&=\sum_{k=1}^n \dfrac{k+2}{k(k+1)(k+3)}\\&=\sum_{k=1}^n\dfrac{(k+2)^2}{k(k+1)(k+2)(k+3)}\\&=-\dfrac{1}{3}\sum_{k=1}^n(k+2)^2\Delta\left[\dfrac{1}{k(k+1)(k+2)}\right]\\&=\left.\dfrac{-(k+2)}{3k(k+1)}\right|_{k=1}^{n+1}+\dfrac{1}{3}\sum_{k=1}^n\dfrac{2k+5}{(k+1)(k+2)(k+3)}\\&=-\dfrac{n+3}{3(n+1)(n+2)}+\dfrac{1}{2}-\dfrac{1}{6}\sum_{k=1}^n\left(\dfrac{4k+11}{(k+2)(k+3)}-\dfrac{4k+7}{(k+1)(k+2)}\right)\\&=-\dfrac{n+3}{3(n+1)(n+2)}+\dfrac{1}{2}-\dfrac{1}{6}\sum_{k=1}^n\Delta\left[\dfrac{4k+7}{(k+1)(k+2)}\right]\\&=-\dfrac{n+3}{3(n+1)(n+2)}+\dfrac{1}{2}-\left.\dfrac{4k+7}{6(k+1)(k+2)}\right|_{k=1}^{n+1}\\&=-\dfrac{n+3}{3(n+1)(n+2)}+\dfrac{1}{2}-\dfrac{4n+11}{6(n+2)(n+3)}+\dfrac{11}{36} \end{align*}$

 

Đến đây thì việc chứng minh đơn giản rồi!







Được gắn nhãn với một hoặc nhiều trong số những từ khóa sau: tuyển tập-sưu tầm.

2 người đang xem chủ đề

0 thành viên, 2 khách, 0 thành viên ẩn danh